Answered step by step
Verified Expert Solution
Link Copied!

Question

1 Approved Answer

will give thumbs up The company produces two different models having different profit margin. Based on the information below, compute total sales mix variance (in

image text in transcribed
image text in transcribed
image text in transcribed
image text in transcribed
will give thumbs up image text in transcribed
The company produces two different models having different profit margin. Based on the information below, compute total sales mix variance (in terms of CM) of the company. Budget Actual Model VC per Selling Price Units Sold Unit Selling Price Vc per Units Sold Unit Model A $120 $60 1,500 $100 $50 1,170 Model B $140 $100 1,500 $150 $110 1,430 Actual Units Sold Actual Budgeted Sales-Mix - Sales-Mix ) models ( Percentage Percentage Budgeted Contribution Margin per Unit Sales- Mix Variance For your convenience, the UCM and sales mix for each product are computed in the following table: Budget Actual UCM Sales mix UCM Sales mix $60 50% $50 45% $40 50% $40 55% a) $2,600 F b) $2.600 U 1 Model B $140 $100 1,500 $150 $110 1,430 Actual Units Sold for all models Actual Budgeted Budgeted Sales-Mix Contribution Sales-Mix ) ~ ( Percentage Percentage Margin per Unit - Sales- Mix Variance For your convenience, the UCM and sales mix for each product are computed in the following table: Budget Actual UCM Sales mix UCM Sales mix $60 50% $50 45% $40 50% $40 55% a) $2,600 F b) $2,600 U Oc) $3,700 U d) $3,700 F Previous Page Next Page Page 26 of 50 Question 27 (3 points) Using the same data in Question 26, compute total sales quantity variance of the company Actual Total units Sold - Budgeted Sales- Budgeted Sales- Budgeted Total Units Sold x Mix Percentages X Contribution Margin= Quantity per Unit Variance a) $22,500 F Ob) $22,500 u c) $20,000 F d) $20,000 u Question 28 (3 points) Using the same data in Question 26, the budgeted market share based on total sales of 37,500 units was 8% (for 3,000 units) when actual total sale volume was 26,000 units in the market. Assuming the budgeted contribution margin per composite unit was $100, compute the market share variance. Market- share Variance Actual Actual Budgeted Market size market in Units x( market share share Budgeted Contribution X Margin per composite unit for budgeted mix a) $43,000 F Ob) $52,000 F O $52,000 d) $43,000 u Question 29 (3 points) Using the same data in Question 26, compute the market size variance. Assume that the budgeted contribution margin per composite unit was $100. Market- size Variance Actual Budgeted Budgeted Budgeted Contribution Market market ) market X Margin per composite unit size size share for budgeted mix = a) $68,000 U Ob) $68,000 F Oc) $92,000 F d) $92,000 U

Step by Step Solution

There are 3 Steps involved in it

Step: 1

blur-text-image

Get Instant Access to Expert-Tailored Solutions

See step-by-step solutions with expert insights and AI powered tools for academic success

Step: 2

blur-text-image

Step: 3

blur-text-image

Ace Your Homework with AI

Get the answers you need in no time with our AI-driven, step-by-step assistance

Get Started

Recommended Textbook for

Financial Accounting A Course For All Majors

Authors: David W. OBryan

1st Edition

1617350958, 978-1617350955

More Books

Students also viewed these Accounting questions

Question

c. A decrease in the price of electricity.

Answered: 1 week ago